Exercices

Exercice

On considère une suite \((u_n)_{n\in \mathbb{N}}\) à cœfficients réels strictement positifs. On considère également \((S_n)_{n\in \mathbb{N}}\) la suite des sommes partielles et, en cas de convergence, \((R_n)_{n\in \mathbb{N}}\) la suite des restes.

1. On suppose que la série \(\sum u_n\) est convergente. Quelle est la nature de la série \(\sum \dfrac{u_n}{S_n^\alpha}\) pour \(\alpha \in \mathbb{R}\) ?

Correction
  • On suppose \(\alpha \geq 0\). Comme la série \(\sum u_n\) est convergente de somme \(S \in \mathbb{R}_+^*\), pour tout \(\varepsilon \in ~]0,S[\), il existe \(N\in \mathbb{N}\) tel que
\[\forall n\geq N, \quad S-\varepsilon \leq S_n \leq S+\varepsilon.\]

Ainsi, comme \(\alpha \geq 0\),

\[\forall n\geq N, \quad 0 \leq \dfrac{u_n}{S_n^\alpha} \leq \dfrac{u_n}{(S-\varepsilon)^\alpha}\]

où le dernier membre est le terme d'un série convergente. Donc, par théorème de comparaison, la série \(\sum \dfrac{u_n}{S_n^\alpha}\) est convergente.

  • On suppose \(\alpha < 0\). On note \(\beta = - \alpha > 0\). Alors
\[\forall n\in \mathbb{N}, \quad \dfrac{u_n}{S_n^\alpha} = u_n S_n^\beta.\]

Puis, comme \(\beta > 0\),

\[\forall n\geq N, \quad 0\leq u_n S_n^\beta \leq u_n S^\beta\]

où le dernier membre est le terme d'un série convergente. Donc, par théorème de comparaison, la série \(\sum \dfrac{u_n}{S_n^\alpha}\) est convergente.

2. On suppose que la série \(\sum u_n\) est divergente.

2.a. Quelle est la nature de la série \(\sum \dfrac{u_n}{S_n}\) ? On pourrer montrer que si la série \(\sum \dfrac{u_n}{S_n}\) est convergente alors la suite \((\ln(S_n))_{n\in \mathbb{N}}\) est convergente.

Correction

On suppose que la série \(\sum \dfrac{u_n}{S_n}\) est convergente. En particulier

\[\dfrac{u_n}{S_n} \underset{n\to +\infty}{\longrightarrow} 0.\]

Ainsi

\[0 < \dfrac{u_n}{S_n} \underset{n\to+\infty}{\sim} - \ln\left(1 - \dfrac{u_n}{S_n} \right) = - \ln \left( \dfrac{S_{n-1}}{S_n} \right) = \ln(S_n) - \ln(S_{n-1}).\]

Donc

\[\ln(S_n) = \sum_{k=0}^n (\ln(S_k) - \ln(S_{k-1})) + \ln(S_0)\]

avec \(\sum (\ln(S_n) - \ln(S_{n-1}))\) terme d'une série convergente d'après ce qui précède. Donc la suite \((\ln(S_n))_{n\in \mathbb{N}}\) est convergente.

2.b. Quelle est la nature de la série \(\sum \dfrac{u_n}{S_n^\alpha}\) pour \(\alpha < 1\) ?

Correction

D'après la question précédente, la série \(\sum \dfrac{u_n}{S_n}\) est divergente. De plus, comme \(\alpha < 1\), nous avons

\[\dfrac{~\dfrac{u_n}{S_n}~}{~\dfrac{u_n}{S_n^\alpha}~} = S_n^{\alpha - 1} \underset{n\to +\infty}{\longrightarrow} 0.\]

Autrement dit

\[\dfrac{u_n}{S_n} \underset{n\to +\infty}{=} o\left( \dfrac{u_n}{S_n^\alpha} \right).\]

Donc, par théorème de comparaison, la série \(\sum \dfrac{u_n}{S_n^\alpha}\) est divergente.

2.c. Quelle est la nature de la série \(\sum \dfrac{u_n}{S_n^\alpha}\) pour \(\alpha > 1\) ? On pourra remarquer que \(\dfrac{u_n}{S_n^\alpha} = \dfrac{S_n - S_{n-1}}{S_n^\alpha}\).

Correction

La suite \((S_n)_{n\in \mathbb{N}}\) est croissante donc

\[0 < \dfrac{u_n}{S_n^\alpha} = \dfrac{S_n - S_{n-1}}{S_n^\alpha} = \int_{S_{n-1}}^{S_n} \dfrac{dx}{S_n^\alpha} \leq \int_{S_{n-1}}^{S_n} \dfrac{dx}{x^\alpha} = \dfrac{1}{\alpha - 1} \left( \dfrac{1}{S_{n-1}^{\alpha - 1}} - \dfrac{1}{S_n^{\alpha - 1}} \right).\]

Donc

\[0\leq \sum_{k=1}^n \dfrac{u_k}{S_k^\alpha} \leq \dfrac{1}{\alpha - 1} \sum_{k=1}^n \left( \dfrac{1}{S_{k-1}^{\alpha - 1}} - \dfrac{1}{S_k^{\alpha - 1}} \right) = \dfrac{1}{\alpha - 1} \left(\dfrac{1}{S_0^{\alpha- 1}} - \dfrac{1}{S_n^{\alpha - 1}} \right) \underset{n\to +\infty}{\longrightarrow} \dfrac{1}{\alpha - 1} \dfrac{1}{S_0^{\alpha- 1}}.\]

Donc la série \(\sum \dfrac{u_k}{S_k^\alpha}\) est convergente.

3. Quelle est la nature de la série \(\sum \dfrac{u_n}{R_{n-1}^\alpha}\) pour \(\alpha \in \mathbb{R}\) dans le cas où la série \(\sum u_n\) est convergente ?

Correction
  • Si \(\alpha \leq 0\) alors, en notant \(\beta = - \alpha\), nous avons, pour tout \(n\in \mathbb{N}^*\),
\[\dfrac{u_n}{R_{n-1}^\alpha} = u_n R_{n-1}^\beta \leq u_n S^\beta\]

où le dernier membre est le terme d'une série convergente. Donc la série \(\sum \dfrac{u_n}{R_{n-1}^\alpha}\) est convergente.

  • Si \(\alpha = 1\) alors pour tout \(n\in \mathbb{N}^*\)
\[\dfrac{u_n}{R_n} = \dfrac{R_{n-1} - R_n}{R_n} = \int_{R_n}^{R_{n-1}} \dfrac{dx}{R_n} \geq \int_{R_n}^{R_{n-1}} \dfrac{dx}{x} = \ln(R_{n-1}) - \ln(R_n).\]

Donc

\[\sum_{k=1}^n \dfrac{u_k}{R_k} \geq \ln(R_0) - \ln(R_n) \underset{n\to +\infty}{\longrightarrow} +\infty.\]

Ainsi la série \(\sum \dfrac{u_n}{R_n}\) diverge. Puis

\[\dfrac{u_n}{R_n} = \dfrac{u_n}{R_{n-1}} \dfrac{1}{1 - \dfrac{u_n}{R_{n-1}}}.\]

Donc si \(\dfrac{u_n}{R_{n-1}} \underset{n\to +\infty}{\longrightarrow} 0\) alors les suites \(\left(\dfrac{u_n}{R_{n-1}}\right)_{n\in \mathbb{N}^*}\) et \(\left(\dfrac{u_n}{R_n}\right)_{n\in \mathbb{N}^*}\) sont équivalentes (et positives). Donc par théorème de comparaison la série \(\sum \dfrac{u_n}{R_{n-1}}\) est divergente. De même si la suite \(\left(\dfrac{u_n}{R_{n-1}}\right)_{n\in \mathbb{N}^*}\) ne converge pas vers \(0\) alors la série \(\sum \dfrac{u_n}{R_{n-1}}\) est grossièrement divergente.

  • Si \(\alpha > 1\) alors, comme \(R_n \underset{n\to +\infty}{\longrightarrow} 0\), il existe un rang \(N\in \mathbb{N}^*\) tel que \(R_n < 1\) pour tout \(n\geq 0\). Donc
\[R_n^\alpha = \exp(\underbrace{\alpha}_{>1} \underbrace{\ln(R_n)}_{< 0}) < \exp(\ln(R_n)) = R_n.\]

Ainsi

\[\dfrac{u_n}{R_n^\alpha} > \dfrac{u_n}{R_n}.\]

Donc la série \(\sum \dfrac{u_n}{R_n^\alpha}\) est divergente.

  • Si \(0< \alpha < 1\) alors pour tout \(n\in \mathbb{N}^*\)
\[\dfrac{u_n}{R_{n-1}^\alpha} = \dfrac{R_{n-1} - R_n}{R_{n-1}^\alpha} = \int_{R_n}^{R_{n-1}} \dfrac{dx}{R_{n-1}^\alpha} \leq \int_{R_n}^{R_{n-1}} \dfrac{dx}{x^\alpha} = \dfrac{1}{1-\alpha} ( R_{n-1}^{1-\alpha}- R_n^{1-\alpha} ).\]

Donc

\[\sum_{k=1}^n \dfrac{u_k}{R_{k-1}^\alpha} \leq \dfrac{1}{1-\alpha} (R_0^{1-\alpha} - R_n^{1-\alpha}) \underset{n\to +\infty}{\longrightarrow} \dfrac{R_0^{1-\alpha}}{1-\alpha}.\]

Donc la série \(\sum \dfrac{u_n}{R_{n-1}^\alpha}\) est convergente.